LSAT and Law School Admissions Forum

Get expert LSAT preparation and law school admissions advice from PowerScore Test Preparation.

 kevin.hussain24
  • Posts: 18
  • Joined: Oct 04, 2019
|
#73417
Hello,

I found this question difficult to find a statement that received support, so I couldn't find what the main point there was, I choice is D. can you please explain why E is correct?

Thank You
Kevin H
 Adam Tyson
PowerScore Staff
  • PowerScore Staff
  • Posts: 5153
  • Joined: Apr 14, 2011
|
#73427
Think of it in the abstract, Kevin - the author is saying "X does not support Y, because you also have to think about Z." The author is trying to prove the first part, that X does not support Y. His premise, supporting that conclusion, is that you also have to take Z into account.

Sometimes it can help to break an argument into abstract bits and pieces to see how they all fit together!

Get the most out of your LSAT Prep Plus subscription.

Analyze and track your performance with our Testing and Analytics Package.